The table shows how the number of sit-ups Marla does each day has changed over time. At this rate, how many sit-ups will she do on Day 12? Explain your steps in solving this problem.see image

The Table Shows How The Number Of Sit-ups Marla Does Each Day Has Changed Over Time. At This Rate, How

Answers

Answer 1

Maria willdo 61 sit-ups on Day 12

Explanation

the table represents a linear function so, we can find the equation of the function and then evaluate for day 12

the equation of a line is given by:

[tex]\begin{gathered} y=mx+b \\ \text{where m is the slope} \\ b\text{ is the y-intercept} \end{gathered}[/tex]

so

Step 1

find the slope of the line

the slope of a line is given by.

[tex]\begin{gathered} \text{slope}=\frac{change\text{ in y }}{\text{change in x}}=\frac{\Delta y}{\Delta x}=\frac{y_2-y_1}{x_2-x_1} \\ \text{where} \\ P1(x_1,y_1) \\ P2(x_2,y_2) \\ \text{are 2 points from the table } \\ or\text{ 2 coordinates ( from the table)} \end{gathered}[/tex]

let

P1(1,17)

P2(4,29)

now, replace

[tex]\begin{gathered} \text{slope}=\frac{y_2-y_1}{x_2-x_1} \\ \text{slope}=\frac{29-17}{4-1}=\frac{12}{3}=4 \\ \text{slope= 4} \end{gathered}[/tex]

Step 2

now,find the equation of the line,use the slope-point formula

[tex]\begin{gathered} y-y_1=m(x-x_1) \\ \text{where m is the slope} \\ \end{gathered}[/tex]

now, replace

[tex]\begin{gathered} y-y_1=m(x-x_1) \\ y-17=4(x-1) \\ y-17=4x-4 \\ y=4x-4+17 \\ y=4x+13 \end{gathered}[/tex]

so,the equation of the lines is

y= 4x+13

Step 3

finally, evaluate for day 12, it is x= 12

so,replace

[tex]\begin{gathered} y=4x+13 \\ y=4(12)+13 \\ y=48+13 \\ y=61 \end{gathered}[/tex]

it means Maria will do 61 sit-ups on Day 12

I hope this helps you


Related Questions

Suppose that there are two types of tickets to a show: advance and same-day. Advance tickets cost $30 and same-day tickets cost $15. For one performance,there were 50 tickets sold in all, and the total amount paid for them was $1275. How many tickets of each type were sold?

Answers

Given:

there are two types of tickets to a show: advance and same-day

Let the number of tickets from the type of Advance = x

And the number of tickets from the type of Same-day = y

there were 50 tickets sold in all

So,

[tex]x+y=50\rightarrow(1)[/tex]

Advance tickets cost $30 and same-day tickets cost $15.

the total amount paid for them was $1275

So,

[tex]30x+15y=1275\rightarrow(2)[/tex]

Solve the equations (1) and (2) to find (x) and (y)

[tex]\begin{gathered} x+y=50\rightarrow(\times-15) \\ 30x+15y=1275 \\ ============= \\ -15x-15y=-750 \\ 30x+15y=1275 \\ ============= \\ 15x=525 \\ x=\frac{525}{15}=35 \\ y=50-x=50-35=15 \end{gathered}[/tex]

So, The answer will be:

The number of tickets from the type of Advance = x = 35

And the number of tickets from the type of Same-day = y = 15

Use the Law of Cosines to determine the indicated angle 0. (Assume a = 65.01, b = 36.38, and c = 42.05. Round your answer to two decimal places.)

Answers

a = 65.01, b = 36.38, and c = 42.05.

And it is required to find the measure of angle 0 which will be the angle B

Using the law of cosine

[tex]\begin{gathered} b^2=a^2+c^2-2\cdot a\cdot c\cdot\cos B \\ \cos B=\frac{a^2+c^2-b^2}{2\cdot a\cdot c}=\frac{65.01^2+42.05^2-36.38^2}{2\cdot65.01\cdot42.05}=0.854 \\ \end{gathered}[/tex][tex]\angle\emptyset=\angle B=\cos ^{-1}0.854=31.31[/tex]

the answer is rounded to two decimals

Carmelo puts $2,200.00 into savings bonds that pay a simple interest rate of 3.4%. How much money will the bonds be worth at the end of 5.5 years? (Find the total worth of the bonds in 5.5 years)

Answers

Let's begin by listing out the information given to us:

Principal (P) = $2,200, Interest rate (r) = 3.4% = 0.034, Time (t) = 5.5 years

[tex]\begin{gathered} I=P\cdot r\cdot t=2200\cdot0.034\cdot5.5 \\ I=\text{ \$414.40} \end{gathered}[/tex]

The bond will be worth the sum of the Principal and the Interest:

[tex]\begin{gathered} P+I=2200+411.40 \\ \Rightarrow\text{ \$}2611.40 \end{gathered}[/tex]

help!!! thanks :))))))

Answers

The length of given sides BC = 38 and EF = 38.

Given:

ΔABC ≅ ΔDEF

BC = x + 30 , EF = 4x + 6

we know that

BC = EF

x + 30 = 4x +  6

30 - 6 = 4x - x

24 = 3x

divide by 3 on both sides

3x/3 = 24/3

x = 8

BC = x + 30

= 8 + 30

= 38

EF = 4x + 6

= 4*8 + 6

= 32+6

= 38

Therefore the length of given sides BC = 38 and EF = 38.

Learn more about the length here:

https://brainly.com/question/8552546

#SPJ1

What is the the musical instrument that is a pair of clash cymbals, originating in the Indian subcontinent, which makes high- pitched percussion sounds?

Answers

SOLUTION:

Step 1:

In this question, we are asked to find the musical instrument that is a pair of clash cymbals, originating in the Indian subcontinent, which makes high- pitched percussion sounds​.

Step 2:

The details of the solution are as follows:

The Taal is a pair of clash cymbals, originating in the Indian subcontinent, which makes high-pitched percussion sounds. In its simplest form, it consists of a pair of small hand cymbals.

usi

how many FULL cases of oil can you get from a 150-gallon oil tank?

Answers

Given

The job is to fill the quart size of bottles, from a full 150 gallon oil tank.

And, the oil is packed into 24 quart's of oil.

To find the number of full cases of oil.

Explanation:

It is given that,

The total amount of oil is 150 gallon.

The number of cases it has to be filled is, 24.

Then, the number of the full cases of oil is,

[tex]\begin{gathered} Number\text{ of full cases of oil}=\frac{150}{24} \\ =\frac{50}{8} \\ =\frac{25}{4} \\ =\frac{24}{4}+\frac{1}{4} \\ =6+\frac{1}{4} \\ =6\frac{1}{4} \end{gathered}[/tex]

Hence, the number of full cases of oil is 6.

options 1. a (-3,50) b (-2,0) c (0,-4) d (1,-6) 2. a (-3,50) b (0,-4) c (-1,-6) d (2,0)3. a (-3,50) b (-2,0) c (0,-4) d (2,0)

Answers

Answer:

The x-intercepts shown in the table are (-2, 0) and (2, 0).

The y-intercept shown in the table is (0, -4)

Explanation:

The x-intercepts are the points where the value of f(x) is 0. Then, these points are (-2, 0) and (2, 0)

Additionally, the y-intercept is the point where the value of x is 0, so the y-intercept is (0, -4).

Then, the answers are:

The x-intercepts shown in the table are (-2, 0) and (2, 0).

The y-intercept shown in the table is (0, -4)

Which is the best estimate of 162% of 79?

Answers

We have to multiply 79 by the percentage in decimal form ( divided by 100)

79 x (162/100) = 79 x 1.62 = 127.98

rounded: 128

The best estimate is 128

Suppose that a regression line for some data transformed with logarithmspredicts that when y equals 8, log(%) will equal 1.603. What does theregression line predict y will equal when y equals 8? Round your answer to thenearest whole number.

Answers

Given the relationship between y and x to be

[tex]y=a^x\text{ ------ equation 1}[/tex]

Take the logarithm of both sides,

[tex]\begin{gathered} \log y=\log ^{}_{}a^x \\ \Rightarrow\log \text{ y = x }\times\text{ log a ---- equation 2} \end{gathered}[/tex]

But when x = 8, log y = 1.603.

Thus, substituting the above values into equation 2, we have

[tex]\begin{gathered} 1.603\text{ = 8 }\times\text{ log a} \\ \text{divide both sides by 8} \\ \log \text{ a= }\frac{1.603}{8} \\ \Rightarrow\log \text{ a =0.2}004 \\ \text{Thus, } \\ a=1.586 \end{gathered}[/tex]

From equation 1,

[tex]\begin{gathered} y=a^x \\ \Rightarrow y=1.586^x\text{ ----- equation 3} \end{gathered}[/tex]

Thus, when x = 8

[tex]\begin{gathered} y=1.586^x \\ y=1.586^8 \\ \Rightarrow y=40.03 \end{gathered}[/tex]

Thus, the value of y will be 40 (to the nearest whole number)

The correct option is D

y=x-8 how would I do it

Answers

to find two points that satisfy the function, you need to give one value and calculate the other one, for example.

I will use x=4 and x=8

when x=4

y=4-8=-4

so one point is (4,-4)

and when x=8

y=8-8=0

so the other point is (8,0)

so you need to graph these points and then plot the line, like this:

Can someone please help me solve this problem number 9

Answers

The perimeter of the room was calculated in question 6, and it is 54 feet.

Since the borders come in 5-yard rolls, let's first convert 54 feet to yards.

Each yard has 3 feet, so we can divide the amount of feet by 3 to get it converted to yards:

[tex]\frac{54}{3}=18[/tex]

Thus, the perimeter of the room is 18 yards. Since each roll has 5 yards,we will need:

[tex]\frac{18}{5}=3.6[/tex]

Abou 3.6 rolls, but since we can only have whole rolls, we will have to approximate it to 4 rolls.

1 Write the missing power of ten.0.04 x 10 = 0.4

Answers

Notice that in the number 0.4, the decimal point appears shifted one place to the right with respect to the number 0.04. When we multiply a number by the power 10^n, the decimal point is shifted n places to the right. Therefore, the power of 10 needed to move the decimal point from 0.04 one place to the right to get 0.4 is 1.

Therefore, the missing power of the base 10 is:

[tex]1[/tex]

So, we can write:

[tex]0.04\times10^1=0.4[/tex]

calculate the surface area of a tetrahedron with four faces and a base of 1 square foot and a height of 0.866 foot

Answers

The surface area is given by:

[tex]SA=B+\frac{1}{2}ph[/tex]

Where:

B = Area of the base

h = height

p = Perimeter of the base

so:

[tex]\begin{gathered} SA=1+\frac{1}{2}(4)(0.866) \\ SA=2.732ft^2 \end{gathered}[/tex]

Find the slope of the line that goes through the points (14,-13) and (2,3).

Answers

Answer

The slope of the line is -4/3

Step-by-step explanation:

Given the following coordinates point

(14, -13) and (2, 3)

Slope = rise / run

rise = y2 - y1

run = x2 - x1

Slope = y2 - y1 / x2 - x1

Let; x1 = 14, y1 = -13, x2 = 2, and y2 = 3

Slope = 3 - (-13) / 2 - 14

Slope = 3 + 13 / - 12

Slope = 16 / -12

Slope = -4/3

Hence, the slope of the line is -4/3

Find the perimeter of each circle. Use 3 for pi.

Answers

Part 1

We need to find the perimeter of a circle with a diameter of 18 inches.

The relation between the perimeter P and the diameter d is given by:

[tex]P=\pi d[/tex]

Since d = 18 inches and we need to use 3 for π, we obtain:

[tex]P=3\cdot18\text{ inches }=54\text{ inches}[/tex]

Therefore, the ribbon needs to be 54 inches long.

Part 2

We need to find the perimeter of a semicircle with a radius of 8 in.

The perimeter of this semicircle is the sum of half the perimeter of the whole circle and the line segment formed by two radii.

The relation between the perimeter P and the radius r of a circle is:

[tex]P=2\pi r[/tex]

Thus, half the perimeter is:

[tex]\frac{P}{2}=\pi r[/tex]

Since we need to use 3 for π and r = 8 in, we obtain:

[tex]\frac{P}{2}=3\cdot8\text{ in }=24\text{ in}[/tex]

And the line segment measures:

[tex]2\cdot8\text{ in }=16\text{ in}[/tex]

Therefore, the perimeter of the calzone is:

[tex]24\text{ in }+16\text{ in }=40\text{ in}[/tex]

Answer: 40 in.

Hey can you help me with my homework also can you tell me the points so I can put them into the graphs

Answers

Step 1

Find the equation of f(x)

[tex]\begin{gathered} The\text{ absolute value function is;} \\ y=a|x-h|+k \end{gathered}[/tex][tex]From\text{ the graph the vertex \lparen h,k\rparen is 3,3}[/tex][tex]\begin{gathered} h=3,k=3 \\ y=1,x=5 \end{gathered}[/tex][tex]1=a|5-3|+3[/tex][tex]\begin{gathered} 1=2a+3 \\ 2a=1-3 \\ 2a=-2 \\ \frac{2a}{2}=-\frac{2}{2} \\ a=-1 \end{gathered}[/tex]

Thus f(x) will be;

[tex]y=-1|x-3|+3[/tex]

Step 2

Find the equation of y= -f(x) then plot the graph

[tex]\begin{gathered} y=-(-1|x-3|+3) \\ y=1\left|x-3\right|-3 \end{gathered}[/tex]

Thus the graph using the points below will look like;

[tex](-4,4),(0,0),(3,-3),(6,0),(8,2)[/tex]

the sum of three consecutive integers is 267.what is that largest interger

Answers

The sum of three consecutive integers is 267. If the first one is a, then the second would be a + 1, while the third would be a + 2. Therefore, you would have;

[tex]\begin{gathered} a+(a+1)+(a+2)=267 \\ a+a+1+a+2=267 \\ 3a+3=267 \\ \text{Subtract 3 from both sides} \\ 3a=264 \\ \text{Divide both sides by 3} \\ a=88 \end{gathered}[/tex]

The largest (third) integer is a + 2, therefore

[tex]\begin{gathered} a+2=88+2 \\ a+2=90 \end{gathered}[/tex]

The largest integer therefore is 90.

Identify the normal equations of an exponential curve.ΣxY = AΣx + BΣx2 and ΣY = nA + BΣxΣxY = AΣx + BΣx2 and ΣY = A + BΣxΣxY = AΣx - BΣx2 and ΣY = nA + BΣxΣxY = AΣx + BΣx2 and ΣY = nA - BΣx

Answers

Given

The normal equations of an exponential curve.



Solution

[tex]The\text{ exponential equation is y=ax}^b[/tex]

taking logarithm on both sides, we get

[tex]\begin{gathered} log10y=log10a+blog10x \\ \\ Y=A+bXwhereY=log10y,A=log10a,X=log10x \end{gathered}[/tex]

which linear in Y,X

So the corresponding normal equations are

[tex]\begin{gathered} ∑Y=nA+b∑X \\ \\ ∑XY=A∑X+b∑X2 \end{gathered}[/tex]

The final answer

Option A

I need help with my math

Answers

[tex]\begin{gathered} 6^{3\text{ }}=\text{ 6 }\times6\times6 \\ 6^2=36 \\ 3^6=\text{ 3}\times3\times3\times3\times3\times3 \end{gathered}[/tex]

Introduction to Chord LengthsINPlace the following expressions so that they can be used to solve for X11 781211 7.8 1211 INN111712

Answers

SOLUTION

We know that the diameter of the circle is 18.8.

Therefore the value of its radius will be.

[tex]\frac{18.8}{2}[/tex]

And we also know that the radius from the diagram is:

[tex]x+4.2[/tex]

So we can equate both equations together to have an idea of what will give us the value of x.

[tex]\begin{gathered} \frac{18.8}{2}=x+4.2 \\ \text{Collect like terms} \\ \frac{18.8}{2}-4.2=x \end{gathered}[/tex]

So going by the above solutions, the answers we will drag into the two boxes will be the 4th expression and the 6th expression

THAT IS:

[tex]\begin{gathered} \frac{18.8}{2} \\ \text{and} \\ -4.2 \end{gathered}[/tex]

Write a recursive definition for the following function. 40, 120,360,1080,3240

Answers

The recursive definition of the given geometric series is  [tex]40 \times (3)^n[/tex]

What is geometric series?

Geometric series are those series in which ratio between the consecutive terms of the series are same.

Here the series is in geometric progression with a common ratio of 3

and first term  40

So the recursive definition of the  given geometric series is [tex]40 \times (3)^n[/tex]

To learn more about Geometric series, refer to the link-

https://brainly.com/question/24643676

#SPJ1

the dealer in a card game draws three cards from a deck of 52 cards and places them face-up on the table select all the correct probabilities

Answers

Explanation:

nCx give us the number of ways in which we can select x cards from a group of n cards.

So, the number of ways in which we can select 3 cards from 52 is:

52C3.

On the other hand, the number of ways to select 3 cards but none of them are kings is 48C3 because there are 48 cards that aren't kings. So:

[tex]P(no\text{ Kings)=}\frac{_{48}C_3}{_{52}C_3}[/tex]

The number of ways to draw 2 fives is: 4C2*48C1

Because the dealer needs to draw 2 cards from the 4 that are fives and 1 card from the other 48 cards. So, P(2 fives) is:

[tex]P(\text{ 2 fives)=}\frac{_4C_2\times_{48}C_1}{_{52}C_3}[/tex]

The number of ways to draw 1 heart and 2 spades is: 13C1*13C2

Because there are 13 heart cards and 13 spades cards. So, P(1 heart and 2 spades) is:

[tex]P(1\text{ Heart and 2 spades) = }\frac{_{13}C_1\times_{13}C_2_{}}{_{52}C_3}[/tex]

Finally, the number of ways to select 4 aces and 1 ten is

which line is steeper y=+2 or y= -7/3x -5

Answers

The inclination of the lines in the coordinate system is given by their slopes, so to determine which line is steeper you have to compare the absolute values of both slopes.

The first equation y=2 has no slope, if you draw it you will see that for any value of x, y doesn't change, it is always 2.

You can also say that the slope of this equation is equal to zero.

For the second equation y=-7/3x-5, the slope is equal to -7/3.

The second equation is steeper than the first one since the absolute value of its slope is greater.

. Kelly makes $475 per week as an assistant I the human resource department of a law firm. What is her annual salary?

Answers

Week salary = $475

In a year there are 52 weeks

Use a rule of three to find the answer

1 week --------------------$475

52 weeks ----------------- x

x = (52 x 475) / 1

x = $24700

Her annual salary is $24700

At an amusement park, the two most popular rollercoasters are the Python and the Vortex. The Python is 212 feet long and the Vortex is 210 feet long. How many times as long is the Python as the Vortex?

Answers

Answer:

About 1.01 times longer

Step-by-step explanation:

we have to divide 212 by 210 since these are the lengths to get about 1.01.

Hopes this helps please mark brainly

Jane has a pre-paid cell phone with A Fee and Fee. She can't remember the exact costs, but her plan has a monthly fee and a charge for each minute of calling time. In June she used 430 minutes and the cost was $227.50. In July she used 780 minutes and the cost was $385.00.

Answers

Given:

the plan of the pre-paid cell phone

The plan has a monthly fee and a charge for each minute

Let the monthly cost = C

and the number of minutes = x

the general equation will be:

C = ax + b

Where (b) is the monthly fee, and (a) is the charge per minute

We will find the values of (a) and (b) using the following:

1) 430 minutes cost $227.50

2) 780 minutes cost $385.00.

So, we have the following equations:

[tex]\begin{gathered} 430a+b=227.5\rightarrow(1) \\ 780a+b=385\rightarrow(2) \end{gathered}[/tex]

Solve the equations, subtract equation (1) from (2) to eliminate (b), and solve for (a):

[tex]\begin{gathered} 780a-430a=385-227.5 \\ 350a=157.5 \\ a=\frac{157.5}{350}=0.45 \end{gathered}[/tex]

Substitute with (a) into equation (1) to find the value of (b)

[tex]\begin{gathered} 430\cdot0.45+b=227.5 \\ 193.5+b=227.5 \\ b=227.5-193.5=34 \end{gathered}[/tex]

So, the equation of the monthly cost will be:

[tex]C=0.45x+34[/tex]

Part (b): When x = 484 minutes, we will find C

so, substitute with (x) into the equation of C

[tex]\begin{gathered} C=0.45\times484+34 \\ C=217.8+34 \\ C=251.8 \end{gathered}[/tex]

So, the answer will be:

A) C = 0.45x + 34

B) $251.8

Given vector v equals open angled bracket negative 11 comma negative 5 close angled bracket comma what are the magnitude and direction of v? Round the magnitude to the thousandths place and the direction to the nearest degree.

Answers

We will begin by finding the magnitude of a vector, denoted |v|.

The formula we can use is

[tex]|v|=\sqrt{a^2+b^2}[/tex]

where a and b represent the vector components. Since we are given the vector <-11,-5>, we will let a be -11 and b is -5.

Substituting those values, we have

[tex]\begin{gathered} |<-11,-1>|=\sqrt{(-11)^2+(-5)^2} \\ \sqrt{121+25} \\ \sqrt{146} \\ \approx12.083 \end{gathered}[/tex]

So far, your answer is either the first option or the second option.

Next, we want to find the direction of the vector. We can use another helpful formula:

[tex]\tan\theta=\frac{b}{a}[/tex]

Substituting our original values for a and b, we have:

[tex]\tan\theta=\frac{-5}{-11}[/tex]

Be careful here! Since the both the a-value and b-value are negative, we are going to be in the third quadrant. After finding our angle (which will be in quadrant 1), we will need to add 180 degrees.

Take the inverse tangent of both sides to get the angle:

[tex]\begin{gathered} \theta=\tan^{-1}(\frac{-5}{-11}) \\ \theta\approx24^{\circ} \end{gathered}[/tex]

We'll add 180 degrees to get our final angle:

[tex]24+180=204[/tex]

Since our final angle is 204 degrees, the correct answer is the second option.

Answer:

12.083; 24°

explanation:

Magnitude of v = sqrt((-11)^2 + (-5)^2)

Direction of v = atan(-5 / -11)

Calculating these values:

Magnitude of v = sqrt(121 + 25) ≈ 12.083 (rounded to the thousandths place)

Direction of v = atan(-5 / -11) ≈ 0.435 radians

Converting radians to degrees:

The direction of v ≈ 0.435 * (180 / π) ≈ 24.881° ≈ 24° (rounded to the nearest degree)

Therefore, the correct answer is 12.083; 24°.

Determine algebraically if f(x)=x^2-8 is a function even, odd, or neither.

Answers

For a function to be even, it has to meet the following condition:

[tex]f(x)=f(-x)[/tex]

To check if the given is an even function, evaluate the function at x and -x:

[tex]\begin{gathered} f(x)=x^2-8 \\ f(-x)=(-x)^2-8=x^2-8 \\ f(x)=f(-x) \end{gathered}[/tex]

It means that the function is even.

For a function to be odd, it has to meet this condition:

[tex]f(-x)=-f(x)[/tex]

We already know the values of f(-x) and f(x) and from this we can state that the function is not odd.

identify the beginning of a sample period for the function

Answers

Given:

[tex]f(t)\text{ = 2csc\lparen t + }\frac{\pi}{4})-1\text{ }[/tex]

The graph of f(t) is shown below:

From the graph, we can see that

[tex]x=\text{ }\frac{\pi}{4}\text{ is a good start for the period of f\lparen t\rparen}[/tex]

Answer: Option D

GIVEN: P(N) = 0.25 and P(R) = 0.6If the probability of P(N R) = 0.15, are N and Rindependent events?a) Yes, because P(N) + P(R) +0.15b) No, because P(N).P(R) +0.15c) Yes, because P(N) X P(R) = 0.15d) Not enough information

Answers

P(N∩R) represents the probability of A and B.

When two events are independent events, the joint probability is calculated by multiplying their individual probabilities.

P(N∩R) For independent events:

[tex]P\mleft(N\cap R\mright)=P(N)\times P(R)[/tex]

Substituting the known values for P(N) and P(R):

[tex]\begin{gathered} P(N\cap R)=0.25\times0.6 \\ P(N\cap R)=0.15 \end{gathered}[/tex]

0.15 is the value of P(N∩R) given by the problem, and since we get the same result using the formula for independent events, we can affirm that N and R independent events.

Answer:

c) Yes, because P(N) X P(R) = 0.15

Other Questions
Conditions for uniform circular motion are constant speed and circular path. (True or false) .Jeremy needs to mail five Christmas packages to his family. The first two weigh 3 pounds each and theother three weigh 2 pounds each. If two-day shipping costs $0.37 per ounce and ground shippingcosts $0.30 per ounce, how much will he save by shipping all of his packages by ground rather thantwo-day? a. Use symbols and proper notation to name the angle shown on thegraph. Write all three correct names. where are the reproductive parts of plants located?leaves,flower,roots, stem Will want to explore these changes in more depth, so we compare two kinds of ways matter changes and how they differ. He recorded his phonics in a science notebook. What are the ways in which matter changes, and how do they differ? Which of the following organisms is would you expect to be most abundant in the tall grass prairie ecosystem? Jillian is looking over a Safety Data Sheet (SDS) for a pesticide at work. She notices it has the exclamation point pictogram shown below. What does this pictogram mean?A red diamond with an exclamation mark inside Chemicals that could cause toxicity to fish and other aquatic life Chronic hazard that comes from long-term exposure Health hazard that is less severe Toxic poisoning after brief exposure write each ratio as a fraction in its simplest form12/10 A hot air balloon is sitting on the ground. Hot air was added causing the balloonto ascend at a rate of 4 feet per second for 60 seconds.A. Use integers to write an expression to determine the location of the airballoon relative to its starting location. WILL GIVE BRAINIEST!!!!! For / (x) = 4x+1 and g(x)=x2-5, find which one of the following options is true when considering the expansion of the binomial expression (x+y)^4?A) The sum of the exponents of each term of the expansion of (x+y)^4 is 5.B) The expansion of (x+y)^4 will yield 4 termsC) The last term of the expansion of (x+y)^4 is y^4D) The coefficients of the expansion of (x+y)^4 are: 1,4,4,1. Andrew is home one winter day studying for an exam. It is lunchtime and he is hungry. Instead of making a sandwich from roast beef in the fridge, he drives to taco bell and spends $5 for 2 tacos, a burrito, and a dr. Pepper. He reasons that he can make $10 per hour cutting lawns so he really saves $5 by going to taco bell rather than preparing his own meal. What's wrong with andrews argument?. what is 80% of 685? Describe the association represented in the graph.no associationstrong, negativestrong positive D weak, negative. pleaneu help us were in class and we don't know the answer guys The population of a small town in central Florida has shown a linear decline in the years 1985-1997. In 1985 the population was 46000 people. In 1997 it was 38080 people. Go to a local poison control website and find an example of the harm exposure to cleaning products can do and what should be done if a person is exposed. Share your findings. Be sure to include a link to the website on which you found the information. A company sells a product for $72 each. The variable costs are $15 per unit and fixed costs are $29,697 per month draw the lewis structure for nh3. what are the approximate bond angles, in degrees, around the n atom?